¿Existe una "raíz cuadrada" de la ecuación de Klein-Gordon que sea diferente de la ecuación de Dirac?

Cuenta la leyenda que Dirac llega a la ecuación de Dirac (todas las ecuaciones están en unidades de Planck C = = 1 en esta publicación):

i γ m m ψ metro ψ = 0
tomando la raíz cuadrada del operador
m m
de la ecuación de Klein-Gordon, que es la versión relativista de la ecuación de Schrödinger:
( m m + metro 2 ) ψ = 0

Sin embargo, ¿es la ecuación de Dirac la única "raíz cuadrada" de la ecuación de Klein-Gordon? ¿Hay alguna otra "raíz cuadrada" de la ecuación de Klein-Gordon?

Resulta que hay una "raíz cuadrada" de la ecuación de Klein-Gordon que es diferente de la ecuación de Dirac:

i γ m m ψ metro mi θ i γ 5 ψ = 0
donde el término de masa de fermión "complejo" está dotado de parte escalar par CP y pseudoescalar impar CP i γ 5 parte:
metro mi θ i γ 5 = metro porque θ + metro i γ 5 pecado θ .

¿Podemos verificar que la ecuación de Dirac "modificada" anterior es de hecho la "raíz cuadrada" de la ecuación de Klein-Gordon? Empecemos con el meollo del asunto:

metro 2 ψ = ( metro mi θ i γ 5 ) ( metro mi θ i γ 5 ) ψ = ( metro mi θ i γ 5 ) ( i γ m m ) ψ = ( i γ m m ) ( metro mi θ i γ 5 ) ψ = ( i γ m m ) ( i γ v v ) ψ = m m ψ .
¡Voila! De hecho, recuperamos la clásica ecuación de Klein-Gordon, sin ningún factor "complejo" divertido. Tenga en cuenta que en la cuarta línea aprovechamos la crucial propiedad anti-conmutación entre γ 5 y γ m .

¿Alguien puede verificar que la derivación anterior sea correcta o incorrecta? ¿O es en realidad lo mismo que la ecuación original de Dirac?


nota añadida

El hecho divertido es que después de una rotación axial del campo de fermiones

ψ mi 1 2 θ i γ 5 ψ .
el término de masa "complejo" en el formato lagrangiano se puede transformar en un término de masa escalar (dejando intacto el término cinético):
metro ψ ¯ mi θ i γ 5 ψ metro ψ ¯ ψ .
En otras palabras, al redefinir el campo de fermiones, la parte impar de CP de la masa metro i γ 5 pecado θ se puede girar de manera efectiva. Dicho esto, sabemos que la masa de fermiones se genera a través del mecanismo de Higgs. La rotación a la masa "real" se puede lograr si solo hay un bosón de Higgs. Si le apetece una fantasía más allá del modelo estándar que involucra múltiples bosones de Higgs, dicha rotación solo puede hacer que un bosón de Higgs CP sea parejo, mientras que deja el otro bosón de Higgs con un CP que rompe el pseudoescalar. i γ 5 fase.

Por supuesto, estos escenarios de bosones de Higgs múltiples generalmente no se consideran en los libros QFT de nivel de entrada. Por lo tanto, la ecuación de Dirac "modificada" generalmente no se menciona en absoluto. Pero, ¿crees que los libros de texto habituales de QFT deberían mencionarlo solo por diversión?

Si trata por separado los espinores diestros y zurdos (como los espinores de Weyl), tendría más libertad en las rotaciones axiales sin invocar explícitamente el pseudoescalar. Esto es exactamente lo que está pasando en las rotaciones relacionadas con CKM. Pero todavía no hay suficiente libertad en el caso de tres generaciones para cancelar todos los pseudoescalares en los parámetros de mezcla de familias, por lo tanto, queda una fase de violación de CP residual en el sector electrodébil.

En pocas palabras, la contraparte de "raíz cuadrada" más general de la ecuación de Klein-Gordon es la ecuación de Dirac con un término de masa "complejo" metro mi θ i γ 5 ψ . La ecuación de masa "real" de Dirac es simplemente un caso especial de θ = 0 .


Más nota añadida:

Hay una cuestión separada del término de masa escalar

metro ψ ¯ ψ
siendo imaginario si reemplaza los componentes del número de Grassmann de ψ y ψ ¯ . Ver detalles aquí .

Una vez hice una pregunta relacionada aquí .
Por supuesto, tiene razón en que, en algunos casos, estos extraños "pseudo espinores de Dirac" no se pueden redefinir para obtener espinores de Dirac ordinarios. Pero en esos casos, la mayoría de los físicos dirían que esto significa tratar de agrupar espinores de Weyl en pares para formar espinores de cuatro componentes es más problemático de lo que vale, y en su lugar trabajarán directamente con los espinores de Weyl.
@knzhou, estuvo de acuerdo, si trata los espinores diestros y zurdos por separado (como los espinores de Weyl), tendría más libertad en rotaciones similares a pseudscalor sin invocar explícitamente el pseudscalor. Esto es exactamente lo que sucede en las rotaciones relacionadas con CKM. Pero todavía no hay suficiente libertad en el caso de tres generaciones para cancelar todos los pseudscalors en los parámetros de mezcla familiar, por lo tanto, queda una fase de violación de CP residual en el sector electrodébil.
@Oбжорoв el triple producto vectorial es un pseudoescalar, es decir, un escalar que cambia de signo bajo una inversión en coordenadas
@MadMax: la derivación es correcta. En realidad, es posible que desee saber que su transformación involucra Exp ( i θ γ 5 ) se conoce como "rotación axial" o "simetría axial" en los libros de texto. Es relevante en el caso sin masa, donde es una simetría de la ecuación de Dirac sin masa (pero es famosamente anómalo en QFT...)
¿Es esto correcto? Ciertamente no es cierto que ( i γ m m metro mi i γ 5 θ ) 2 = ( i γ m m metro mi i γ 5 θ ) ( i γ m m metro mi i γ 5 θ ) da la ecuación de Dirac porque, por ejemplo, los términos cruzados no se cancelan. Parece que estás tratando de sacar la raíz cuadrada compleja usando un es decir ( i γ m m metro mi i γ 5 θ ) ( i γ m m metro mi i γ 5 θ ) pero esto no parece tener sentido ya que hace que actúe a la izquierda, no a la derecha...
Ciertamente no puedes estar haciendo esto simplemente tomando el conjugado complejo real ( i γ m m metro mi i γ 5 θ ) ( i γ m m metro mi i γ 5 θ ) porque { γ m , γ v } 2 η m v cierto, de hecho tendrías que hacer m v γ m γ v = 1 2 m v ( γ m γ v + γ v γ m ) lo que es aún peor, entonces, ¿tiene sentido esto? En cualquier caso, esta pregunta nunca debería haberse cerrado.
Esto parece estar relacionado con physics.stackexchange.com/q/156124/9290

Respuestas (2)

Parece haber cierta confusión en los comentarios, así que aquí hay una respuesta completa.

En primer lugar conjunto η = ( , + , + , + ) y

γ m γ v + γ v γ m = 2 η m v , ( γ 5 ) 2 = 1 , γ 5 γ m + γ m γ 5 = 0 .

Considere primero la ecuación estándar de Dirac:

( m γ m + metro ) ψ = 0 .
Encontramos
0 = ( m γ m + metro ) ( m γ m + metro ) ψ = η m v m v ψ + metro 2 ψ .
Así que no tenemos literalmente una raíz cuadrada. Necesita un cambio de signo para matar los términos cruzados, de manera similar a cómo X 2 + y 2 = ( X + i y ) ( X i y ) . Dado que aquí estamos usando la convención mayormente positiva, reconocemos la ecuación de Klein-Gordon en la última igualdad aquí. (Ver también Peskin & Schroeder ch 3.2, aunque usan la terrible convención mayormente menos).

Qué pasa

( m γ m + metro Exp ( i θ γ 5 ) ) ψ = 0 ?
¿Es esta una "raíz cuadrada" en el sentido anterior? Seguro; todas las soluciones de lo anterior también resuelven la ecuación de KG:
0 = ( m γ m + metro Exp ( i θ γ 5 ) ) ( m γ m + metro Exp ( i θ γ 5 ) ) ψ = ( η m v m v + metro mi i θ γ 5 m γ m metro m γ m mi + i θ γ 5 + metro 2 ) ψ = η m v m v ψ + metro 2 ψ .
Desde γ 5 anti desplazamientos con γ m , los términos cruzados se cancelan.

Esto es esencialmente lo mismo que la derivación dada en la pregunta (con la que estoy de acuerdo).

Como mencioné en mi comentario anterior, la rotación axial que elimina el término de masa del que trata la pregunta es más popular que esta ecuación de Dirac modificada, aunque no sea por otra razón que

  1. La rotación axial es una simetría en el caso sin masa, y
  2. En el caso masivo, generalmente puede redefinir sus fermiones con la rotación axial para matar el término de masa metro Exp ( i θ γ 5 ) en favor de metro .

EDITAR: claramente puede clasificar todas las "ecuaciones de Dirac" similares tabulando matrices que (anti) conmutan con todas γ m . Por supuesto que es material de libro de texto. Mi recuerdo es que el argumento del lema de Schur muestra que todos están en el lapso de 1 y γ 5 (en cualquier dimensión).

La unicidad es un tema de investigación relativamente más amplio: necesitamos verificar qué es aplicable en qué contexto: eche un vistazo a esto: La raíz cuadrada del operador pseudodiferencial de la ecuación de Klein-Gordon - Claus Lammerzahla, 1993, ZARM . Ver también: https://www.zarm.uni-bremen.de

¿Te refieres a este papel? @ zarm.uni-bremen.de/uploads/tx_sibibtex/… . Puede ser mejor poner el enlace en su respuesta.
Ok, lo pondré ahí en algún momento...